Sie sind auf Seite 1von 7

PAB3053 Reservoir Modeling and Simulation Solutions to Assignment # 1

Dr. Noaman El-Khatib1


1 Department

of Petroleum Engineering, Universiti Teknologi Petronas

Due: August 8, 2009

Solution to Problem # 1

Problem: Consider a linear ow system with Area = 25 ft2 . End point A is 5 ft higher than end point B, and the distance between end points is 15 ft. The permeability is 1 Darcy. Suppose the system contains oil with viscosity = 0.8 cp, gravity=350 API (0 =0.85), and FVF = 1.0 RB/STB. If the end point pressures are PA =PB =20 psia, is there ow? If so, how much and in what direction? Answer: See gure below,

Figure 1: Scenario of Problem # 1

Recall that, = P + 0.433h Let point B be the reference point, thus (1)

B = 20 psia A = 20 + (0.433)(0.85)(5) = 20 + 1.94025 = 21.94025 Since A > B , ow is A B. The ow rate q is, 1.127KA 0 L (1.127)(1)(25)(1.94025) = (1)(0.8)(15) = 4.32 STB/day

(2)

(3)

q =

(4)

Solution to Problem # 2

Problem: Using the data in Exercise 7-1, calculate the pressure PB that would prevent uid ow. Answer: To prevent ow, A = B = 21.94025, thus PB = B = 21.94025 psia (5)

Solution to Problem # 3

Problem: Suppose the pressure P1 of a water-bearing formation at depth z1 10,000 ft is 4000 psia. If the pressure gradient for water is 0.433 psia/ft, calculate the pressure P2 at depth Z2 = 11,000 ft. Calculate the phase potential 1 and 2 at depths Z1 and Z2 respectively. Answer: The scenario can be described in Figure 2.

Figure 2: Scenario of Problem # 3 At 11,000 ft,

P2 = P1 + 0.433h = 4000 + (0.433)(1000) = 4433 psia Assume reference point at z=0. Therefore, 1 = P1 (0.433)(40, 000) = 4000 4330 = 330 psia 2 = P2 (0.433)(11, 000) = 4433 4763 = 330 psia

(6)

(7)

(8)

4
z

Solution to Problem # 4
P z

Problem: Use the pressures and potentials in Problem 3 to estimate the derivatives

and

Answer: Here,

P z

4433 4000 1000 0.433 330 (330) z 1000 0

(9)

(10)

Solution to Problem # 5

Problem: For a core with the following properties: Permeability = 1000 mD Area = 5 cm2 = Length = 10 cm Water with viscosity of 1 cp. And density of 1 gm/cm3 is passed through the core at inlet pressure of 20 psia and outlet pressure 15 psia. Calculate the ow rate for the following cases: 1- The ow is horizontal 2- The ow is inclined 300 upward 3- The ow is inclined 300 downward 4- The ow is vertical upward 5- The ow is vertical downward Answer: The scenario can be shown in the following gure,

Figure 3: Scenario of Problem # 5 We will start with Darcys law in terms of ow rate, q= 0.001127KA (1 2 ) L 4 (11)

Known parameters are: K = 1 darcy = 1000 md A = 5 cm2 L = 10 cm = 1 cp = 1 gm/cm3 Pinlet = P1 = 20 psia Poutlet = P2 = 15 psia = vary by cases First we would write the potentials given zdatum = z0 = z1

1 = P1 0.433z1 1 = P2 + 0.433z2 Thus, given z1 = z1 z0 = 0, 0.001127KA (1 2 ) L 0.001127KA 1 2 L 0.001127KA P1 P2 z2 0.433 L L 0.001127KA P1 P2 0.433 sin L 20 15 5.635 0.433(1) sin 10 5.635(0.5 0.433 sin )

(12) (13)

q = = = = = =

(14)

Based on equation (14), the ow rate can be calculated based on the following variations, Table 1: case 1 2 3 4 5 values in terms of cases sin q 00 0.0 2.8175 300 0.5 1.597522 0 -30 -0.5 4.037477 900 1.0 0.377545 0 -90 1.0 5.257455

Solution to Problem # 6

Problem: Oil with 2 cp. Viscosity and 450 API is owing in a core with length 200 ft and cross=sectional area 50 ft2 . The core is inclined 300 from the horizontal and the higher end of the core is kept at atmospheric pressure. Calculate the ow rate in Bbl/day in the core for the following cases: a. Inlet Pressure = 200 psia b. Inlet Pressure = 120 psia c. Inlet Pressure = 50 psia Answer: The scenario can be shown on the following gure,

Figure 4: Scenario of Problem # 6 To start with, we calculate the specic gravity sp.gr = Having
dp dx

141.5 141.5 = 0.8 131.5 + API 131.5 + 45

(15)

in

psi , ft

K in Darcy, in cp, thus we can write Darcys law as follow: 1.127K dp + 0.433 sin dx

v =

(16)

(1.127)(0.5) P 3 = + (0.433)(0.8)( ) 2 200 2 = 0.28175 P 0.3 200 (17)

The expression for P, given absolute pressure Pa can be written as follows: P = Pin Pa = Pin 14.7 (18)

Thus, we can come up with the following table: Table 2: Results for Problem # 4 P P 0.3 v Pin (psi) P 200 200 200 185.3 0.9265 0.6265 0.1765 120 105.3 0.5265 0.2265 0.0638 50 35.3 0.1765 -0.1235 -0.0348

case i ii iii

Note: In case (iii), v is negative, thus the ow dips downward (reversed). ========== All Ends Here

Das könnte Ihnen auch gefallen